Let $p$ be a prime of the form $3k+2$. Show that if $x^3 equiv 1 pmod p$ then $x equiv 1 pmod p$.Prove that $x^2 equiv -1 pmod p$ has no solutions if prime $p equiv 3 pmod 4$.Show that if $R_n$ is prime then $n$ must be prime.Show that $16$ is a perfect $8$th power modulo $p$ for any prime number $p$Show that $10^n(p-1)equiv 1pmod! 9p$ for $pge 7$Number Theory: $m^2equiv 1pmodp^n$If $a^p-1 equiv 1 pmod p$ and then $a^fracp-12 equiv 1$ or $p-1 pmod p$?Show that $a^i equiv a^j pmodm$ iff $i equiv j pmodd$Find solution of $x^p+1equiv1pmodp^q$, where p and q are primeProve that if $aequiv b pmod p^2-p$, then $a^aequiv b^b pmodp$ where $p$ is any prime and $a$ and $b$ are nonzero integers.Elementary Number Theory: Show that $3^10equiv 1 pmod11^2$.

How do you justify more code being written by following clean code practices?

is this saw blade faulty?

Imaginary part of expression too difficult to calculate

Isn't the word "experience" wrongly used in this context?

Does convergence of polynomials imply that of its coefficients?

PTIJ: Which Dr. Seuss books should one obtain?

How to test the sharpness of a knife?

Do people actually use the word "kaputt" in conversation?

Justification failure in beamer enumerate list

Why doesn't the chatan sign the ketubah?

What are the rules for concealing thieves' tools (or items in general)?

How are passwords stolen from companies if they only store hashes?

Turning a hard to access nut?

How can an organ that provides biological immortality be unable to regenerate?

How to find the largest number(s) in a list of elements, possibly non-unique?

Does the Shadow Magic sorcerer's Eyes of the Dark feature work on all Darkness spells or just his/her own?

TDE Master Key Rotation

Emojional cryptic crossword

CLI: Get information Ubuntu releases

Exit shell with shortcut (not typing exit) that closes session properly

Why are there no stars visible in cislunar space?

Have the tides ever turned twice on any open problem?

Is this Pascal's Matrix?

Why is this tree refusing to shed its dead leaves?



Let $p$ be a prime of the form $3k+2$. Show that if $x^3 equiv 1 pmod p$ then $x equiv 1 pmod p$.


Prove that $x^2 equiv -1 pmod p$ has no solutions if prime $p equiv 3 pmod 4$.Show that if $R_n$ is prime then $n$ must be prime.Show that $16$ is a perfect $8$th power modulo $p$ for any prime number $p$Show that $10^n(p-1)equiv 1pmod! 9p$ for $pge 7$Number Theory: $m^2equiv 1pmodp^n$If $a^p-1 equiv 1 pmod p$ and then $a^fracp-12 equiv 1$ or $p-1 pmod p$?Show that $a^i equiv a^j pmodm$ iff $i equiv j pmodd$Find solution of $x^p+1equiv1pmodp^q$, where p and q are primeProve that if $aequiv b pmod p^2-p$, then $a^aequiv b^b pmodp$ where $p$ is any prime and $a$ and $b$ are nonzero integers.Elementary Number Theory: Show that $3^10equiv 1 pmod11^2$.













3












$begingroup$



Let $p$ be a prime of the form $3k+2$. Show that if $x^3 equiv 1 pmod p$ then $x equiv 1 pmod p$.




What seems like and is probably an incredibly easy question and I'm struggling to get anywhere.



I've tried showing that since $p$ is prime and $p|x^3-1$ then either $p|x-1$ or $p|x^2+x+1$ so we're done if we can show that the latter case can never come about, however I'm struggling to show this. I'm doubtful this is the correct approach but I can't see anything else.



Any help is appreciated.



Thank you.










share|cite|improve this question











$endgroup$
















    3












    $begingroup$



    Let $p$ be a prime of the form $3k+2$. Show that if $x^3 equiv 1 pmod p$ then $x equiv 1 pmod p$.




    What seems like and is probably an incredibly easy question and I'm struggling to get anywhere.



    I've tried showing that since $p$ is prime and $p|x^3-1$ then either $p|x-1$ or $p|x^2+x+1$ so we're done if we can show that the latter case can never come about, however I'm struggling to show this. I'm doubtful this is the correct approach but I can't see anything else.



    Any help is appreciated.



    Thank you.










    share|cite|improve this question











    $endgroup$














      3












      3








      3





      $begingroup$



      Let $p$ be a prime of the form $3k+2$. Show that if $x^3 equiv 1 pmod p$ then $x equiv 1 pmod p$.




      What seems like and is probably an incredibly easy question and I'm struggling to get anywhere.



      I've tried showing that since $p$ is prime and $p|x^3-1$ then either $p|x-1$ or $p|x^2+x+1$ so we're done if we can show that the latter case can never come about, however I'm struggling to show this. I'm doubtful this is the correct approach but I can't see anything else.



      Any help is appreciated.



      Thank you.










      share|cite|improve this question











      $endgroup$





      Let $p$ be a prime of the form $3k+2$. Show that if $x^3 equiv 1 pmod p$ then $x equiv 1 pmod p$.




      What seems like and is probably an incredibly easy question and I'm struggling to get anywhere.



      I've tried showing that since $p$ is prime and $p|x^3-1$ then either $p|x-1$ or $p|x^2+x+1$ so we're done if we can show that the latter case can never come about, however I'm struggling to show this. I'm doubtful this is the correct approach but I can't see anything else.



      Any help is appreciated.



      Thank you.







      number-theory prime-numbers modular-arithmetic






      share|cite|improve this question















      share|cite|improve this question













      share|cite|improve this question




      share|cite|improve this question








      edited Mar 13 at 4:33









      Rócherz

      2,9863821




      2,9863821










      asked Aug 27 '16 at 19:20









      Aka_aka_aka_akAka_aka_aka_ak

      935415




      935415




















          3 Answers
          3






          active

          oldest

          votes


















          2












          $begingroup$

          Hint $, rm mod, p!:, 1 equiv x^largecolor#0a0 p-1 equiv x^large 1+3kequiv x(color#c00x^large 3)^large k equiv x, $ by $,color#c00x^large 3equiv 1. $ QED



          Said conceptually: $ $ the order of $,x,$ is $,1,$ since it divides the coprimes $,color#c003,$ and $,color#0a0p!-!1 = 1+3k$






          share|cite|improve this answer











          $endgroup$








          • 1




            $begingroup$
            That's a nice way to solve it.
            $endgroup$
            – quid
            Aug 28 '16 at 0:43


















          1












          $begingroup$

          The approach is fine.



          You can exclude the latter if you can show that $x^2 + x + 1 $ has no root modulo $p$. This is equivalent to its discriminant not being a quadratic residue modulo $p$. And this in turn follows by the congruence condition on $p$.






          share|cite|improve this answer









          $endgroup$












          • $begingroup$
            Let me know if you want more computational details.
            $endgroup$
            – quid
            Aug 27 '16 at 19:26










          • $begingroup$
            Ok thanks, I'll take another look in a minute, this will likely be enough, thank you.
            $endgroup$
            – Aka_aka_aka_ak
            Aug 27 '16 at 19:28


















          1












          $begingroup$

          If $p$ is prime and $pmid x^3-1=(x-1)left(x^2+x+1right)$, then by Euclid's lemma either $pmid x-1$ or $pmid x^2+x+1$.



          For contradiction, let $pmid x^2+x+1$ and $pequiv 2pmod 3$.



          If $p=2$, then $2=pmid x^2+x+1=x(x+1)+1$, which is odd for all $xinmathbb Z$, contradiction.



          If $p$ is odd, then $pmid 4left(x^2+x+1right)=(2x+1)^2+3$, so $(2x+1)^2equiv -3pmod p$, so $-3$ is a quadratic residue mod $p$, but this contradicts Quadratic Reciprocity.






          share|cite|improve this answer









          $endgroup$












            Your Answer





            StackExchange.ifUsing("editor", function ()
            return StackExchange.using("mathjaxEditing", function ()
            StackExchange.MarkdownEditor.creationCallbacks.add(function (editor, postfix)
            StackExchange.mathjaxEditing.prepareWmdForMathJax(editor, postfix, [["$", "$"], ["\\(","\\)"]]);
            );
            );
            , "mathjax-editing");

            StackExchange.ready(function()
            var channelOptions =
            tags: "".split(" "),
            id: "69"
            ;
            initTagRenderer("".split(" "), "".split(" "), channelOptions);

            StackExchange.using("externalEditor", function()
            // Have to fire editor after snippets, if snippets enabled
            if (StackExchange.settings.snippets.snippetsEnabled)
            StackExchange.using("snippets", function()
            createEditor();
            );

            else
            createEditor();

            );

            function createEditor()
            StackExchange.prepareEditor(
            heartbeatType: 'answer',
            autoActivateHeartbeat: false,
            convertImagesToLinks: true,
            noModals: true,
            showLowRepImageUploadWarning: true,
            reputationToPostImages: 10,
            bindNavPrevention: true,
            postfix: "",
            imageUploader:
            brandingHtml: "Powered by u003ca class="icon-imgur-white" href="https://imgur.com/"u003eu003c/au003e",
            contentPolicyHtml: "User contributions licensed under u003ca href="https://creativecommons.org/licenses/by-sa/3.0/"u003ecc by-sa 3.0 with attribution requiredu003c/au003e u003ca href="https://stackoverflow.com/legal/content-policy"u003e(content policy)u003c/au003e",
            allowUrls: true
            ,
            noCode: true, onDemand: true,
            discardSelector: ".discard-answer"
            ,immediatelyShowMarkdownHelp:true
            );



            );













            draft saved

            draft discarded


















            StackExchange.ready(
            function ()
            StackExchange.openid.initPostLogin('.new-post-login', 'https%3a%2f%2fmath.stackexchange.com%2fquestions%2f1905547%2flet-p-be-a-prime-of-the-form-3k2-show-that-if-x3-equiv-1-pmod-p-then%23new-answer', 'question_page');

            );

            Post as a guest















            Required, but never shown

























            3 Answers
            3






            active

            oldest

            votes








            3 Answers
            3






            active

            oldest

            votes









            active

            oldest

            votes






            active

            oldest

            votes









            2












            $begingroup$

            Hint $, rm mod, p!:, 1 equiv x^largecolor#0a0 p-1 equiv x^large 1+3kequiv x(color#c00x^large 3)^large k equiv x, $ by $,color#c00x^large 3equiv 1. $ QED



            Said conceptually: $ $ the order of $,x,$ is $,1,$ since it divides the coprimes $,color#c003,$ and $,color#0a0p!-!1 = 1+3k$






            share|cite|improve this answer











            $endgroup$








            • 1




              $begingroup$
              That's a nice way to solve it.
              $endgroup$
              – quid
              Aug 28 '16 at 0:43















            2












            $begingroup$

            Hint $, rm mod, p!:, 1 equiv x^largecolor#0a0 p-1 equiv x^large 1+3kequiv x(color#c00x^large 3)^large k equiv x, $ by $,color#c00x^large 3equiv 1. $ QED



            Said conceptually: $ $ the order of $,x,$ is $,1,$ since it divides the coprimes $,color#c003,$ and $,color#0a0p!-!1 = 1+3k$






            share|cite|improve this answer











            $endgroup$








            • 1




              $begingroup$
              That's a nice way to solve it.
              $endgroup$
              – quid
              Aug 28 '16 at 0:43













            2












            2








            2





            $begingroup$

            Hint $, rm mod, p!:, 1 equiv x^largecolor#0a0 p-1 equiv x^large 1+3kequiv x(color#c00x^large 3)^large k equiv x, $ by $,color#c00x^large 3equiv 1. $ QED



            Said conceptually: $ $ the order of $,x,$ is $,1,$ since it divides the coprimes $,color#c003,$ and $,color#0a0p!-!1 = 1+3k$






            share|cite|improve this answer











            $endgroup$



            Hint $, rm mod, p!:, 1 equiv x^largecolor#0a0 p-1 equiv x^large 1+3kequiv x(color#c00x^large 3)^large k equiv x, $ by $,color#c00x^large 3equiv 1. $ QED



            Said conceptually: $ $ the order of $,x,$ is $,1,$ since it divides the coprimes $,color#c003,$ and $,color#0a0p!-!1 = 1+3k$







            share|cite|improve this answer














            share|cite|improve this answer



            share|cite|improve this answer








            edited Aug 27 '16 at 21:34

























            answered Aug 27 '16 at 21:19









            Bill DubuqueBill Dubuque

            212k29195654




            212k29195654







            • 1




              $begingroup$
              That's a nice way to solve it.
              $endgroup$
              – quid
              Aug 28 '16 at 0:43












            • 1




              $begingroup$
              That's a nice way to solve it.
              $endgroup$
              – quid
              Aug 28 '16 at 0:43







            1




            1




            $begingroup$
            That's a nice way to solve it.
            $endgroup$
            – quid
            Aug 28 '16 at 0:43




            $begingroup$
            That's a nice way to solve it.
            $endgroup$
            – quid
            Aug 28 '16 at 0:43











            1












            $begingroup$

            The approach is fine.



            You can exclude the latter if you can show that $x^2 + x + 1 $ has no root modulo $p$. This is equivalent to its discriminant not being a quadratic residue modulo $p$. And this in turn follows by the congruence condition on $p$.






            share|cite|improve this answer









            $endgroup$












            • $begingroup$
              Let me know if you want more computational details.
              $endgroup$
              – quid
              Aug 27 '16 at 19:26










            • $begingroup$
              Ok thanks, I'll take another look in a minute, this will likely be enough, thank you.
              $endgroup$
              – Aka_aka_aka_ak
              Aug 27 '16 at 19:28















            1












            $begingroup$

            The approach is fine.



            You can exclude the latter if you can show that $x^2 + x + 1 $ has no root modulo $p$. This is equivalent to its discriminant not being a quadratic residue modulo $p$. And this in turn follows by the congruence condition on $p$.






            share|cite|improve this answer









            $endgroup$












            • $begingroup$
              Let me know if you want more computational details.
              $endgroup$
              – quid
              Aug 27 '16 at 19:26










            • $begingroup$
              Ok thanks, I'll take another look in a minute, this will likely be enough, thank you.
              $endgroup$
              – Aka_aka_aka_ak
              Aug 27 '16 at 19:28













            1












            1








            1





            $begingroup$

            The approach is fine.



            You can exclude the latter if you can show that $x^2 + x + 1 $ has no root modulo $p$. This is equivalent to its discriminant not being a quadratic residue modulo $p$. And this in turn follows by the congruence condition on $p$.






            share|cite|improve this answer









            $endgroup$



            The approach is fine.



            You can exclude the latter if you can show that $x^2 + x + 1 $ has no root modulo $p$. This is equivalent to its discriminant not being a quadratic residue modulo $p$. And this in turn follows by the congruence condition on $p$.







            share|cite|improve this answer












            share|cite|improve this answer



            share|cite|improve this answer










            answered Aug 27 '16 at 19:26









            quidquid

            37.2k95193




            37.2k95193











            • $begingroup$
              Let me know if you want more computational details.
              $endgroup$
              – quid
              Aug 27 '16 at 19:26










            • $begingroup$
              Ok thanks, I'll take another look in a minute, this will likely be enough, thank you.
              $endgroup$
              – Aka_aka_aka_ak
              Aug 27 '16 at 19:28
















            • $begingroup$
              Let me know if you want more computational details.
              $endgroup$
              – quid
              Aug 27 '16 at 19:26










            • $begingroup$
              Ok thanks, I'll take another look in a minute, this will likely be enough, thank you.
              $endgroup$
              – Aka_aka_aka_ak
              Aug 27 '16 at 19:28















            $begingroup$
            Let me know if you want more computational details.
            $endgroup$
            – quid
            Aug 27 '16 at 19:26




            $begingroup$
            Let me know if you want more computational details.
            $endgroup$
            – quid
            Aug 27 '16 at 19:26












            $begingroup$
            Ok thanks, I'll take another look in a minute, this will likely be enough, thank you.
            $endgroup$
            – Aka_aka_aka_ak
            Aug 27 '16 at 19:28




            $begingroup$
            Ok thanks, I'll take another look in a minute, this will likely be enough, thank you.
            $endgroup$
            – Aka_aka_aka_ak
            Aug 27 '16 at 19:28











            1












            $begingroup$

            If $p$ is prime and $pmid x^3-1=(x-1)left(x^2+x+1right)$, then by Euclid's lemma either $pmid x-1$ or $pmid x^2+x+1$.



            For contradiction, let $pmid x^2+x+1$ and $pequiv 2pmod 3$.



            If $p=2$, then $2=pmid x^2+x+1=x(x+1)+1$, which is odd for all $xinmathbb Z$, contradiction.



            If $p$ is odd, then $pmid 4left(x^2+x+1right)=(2x+1)^2+3$, so $(2x+1)^2equiv -3pmod p$, so $-3$ is a quadratic residue mod $p$, but this contradicts Quadratic Reciprocity.






            share|cite|improve this answer









            $endgroup$

















              1












              $begingroup$

              If $p$ is prime and $pmid x^3-1=(x-1)left(x^2+x+1right)$, then by Euclid's lemma either $pmid x-1$ or $pmid x^2+x+1$.



              For contradiction, let $pmid x^2+x+1$ and $pequiv 2pmod 3$.



              If $p=2$, then $2=pmid x^2+x+1=x(x+1)+1$, which is odd for all $xinmathbb Z$, contradiction.



              If $p$ is odd, then $pmid 4left(x^2+x+1right)=(2x+1)^2+3$, so $(2x+1)^2equiv -3pmod p$, so $-3$ is a quadratic residue mod $p$, but this contradicts Quadratic Reciprocity.






              share|cite|improve this answer









              $endgroup$















                1












                1








                1





                $begingroup$

                If $p$ is prime and $pmid x^3-1=(x-1)left(x^2+x+1right)$, then by Euclid's lemma either $pmid x-1$ or $pmid x^2+x+1$.



                For contradiction, let $pmid x^2+x+1$ and $pequiv 2pmod 3$.



                If $p=2$, then $2=pmid x^2+x+1=x(x+1)+1$, which is odd for all $xinmathbb Z$, contradiction.



                If $p$ is odd, then $pmid 4left(x^2+x+1right)=(2x+1)^2+3$, so $(2x+1)^2equiv -3pmod p$, so $-3$ is a quadratic residue mod $p$, but this contradicts Quadratic Reciprocity.






                share|cite|improve this answer









                $endgroup$



                If $p$ is prime and $pmid x^3-1=(x-1)left(x^2+x+1right)$, then by Euclid's lemma either $pmid x-1$ or $pmid x^2+x+1$.



                For contradiction, let $pmid x^2+x+1$ and $pequiv 2pmod 3$.



                If $p=2$, then $2=pmid x^2+x+1=x(x+1)+1$, which is odd for all $xinmathbb Z$, contradiction.



                If $p$ is odd, then $pmid 4left(x^2+x+1right)=(2x+1)^2+3$, so $(2x+1)^2equiv -3pmod p$, so $-3$ is a quadratic residue mod $p$, but this contradicts Quadratic Reciprocity.







                share|cite|improve this answer












                share|cite|improve this answer



                share|cite|improve this answer










                answered Aug 27 '16 at 19:34









                user236182user236182

                12k11333




                12k11333



























                    draft saved

                    draft discarded
















































                    Thanks for contributing an answer to Mathematics Stack Exchange!


                    • Please be sure to answer the question. Provide details and share your research!

                    But avoid


                    • Asking for help, clarification, or responding to other answers.

                    • Making statements based on opinion; back them up with references or personal experience.

                    Use MathJax to format equations. MathJax reference.


                    To learn more, see our tips on writing great answers.




                    draft saved


                    draft discarded














                    StackExchange.ready(
                    function ()
                    StackExchange.openid.initPostLogin('.new-post-login', 'https%3a%2f%2fmath.stackexchange.com%2fquestions%2f1905547%2flet-p-be-a-prime-of-the-form-3k2-show-that-if-x3-equiv-1-pmod-p-then%23new-answer', 'question_page');

                    );

                    Post as a guest















                    Required, but never shown





















































                    Required, but never shown














                    Required, but never shown












                    Required, but never shown







                    Required, but never shown

































                    Required, but never shown














                    Required, but never shown












                    Required, but never shown







                    Required, but never shown







                    Popular posts from this blog

                    Lowndes Grove History Architecture References Navigation menu32°48′6″N 79°57′58″W / 32.80167°N 79.96611°W / 32.80167; -79.9661132°48′6″N 79°57′58″W / 32.80167°N 79.96611°W / 32.80167; -79.9661178002500"National Register Information System"Historic houses of South Carolina"Lowndes Grove""+32° 48' 6.00", −79° 57' 58.00""Lowndes Grove, Charleston County (260 St. Margaret St., Charleston)""Lowndes Grove"The Charleston ExpositionIt Happened in South Carolina"Lowndes Grove (House), Saint Margaret Street & Sixth Avenue, Charleston, Charleston County, SC(Photographs)"Plantations of the Carolina Low Countrye

                    random experiment with two different functions on unit interval Announcing the arrival of Valued Associate #679: Cesar Manara Planned maintenance scheduled April 23, 2019 at 00:00UTC (8:00pm US/Eastern)Random variable and probability space notionsRandom Walk with EdgesFinding functions where the increase over a random interval is Poisson distributedNumber of days until dayCan an observed event in fact be of zero probability?Unit random processmodels of coins and uniform distributionHow to get the number of successes given $n$ trials , probability $P$ and a random variable $X$Absorbing Markov chain in a computer. Is “almost every” turned into always convergence in computer executions?Stopped random walk is not uniformly integrable

                    How should I support this large drywall patch? Planned maintenance scheduled April 23, 2019 at 00:00UTC (8:00pm US/Eastern) Announcing the arrival of Valued Associate #679: Cesar Manara Unicorn Meta Zoo #1: Why another podcast?How do I cover large gaps in drywall?How do I keep drywall around a patch from crumbling?Can I glue a second layer of drywall?How to patch long strip on drywall?Large drywall patch: how to avoid bulging seams?Drywall Mesh Patch vs. Bulge? To remove or not to remove?How to fix this drywall job?Prep drywall before backsplashWhat's the best way to fix this horrible drywall patch job?Drywall patching using 3M Patch Plus Primer